LSAT and Law School Admissions Forum

Get expert LSAT preparation and law school admissions advice from PowerScore Test Preparation.

 mbeatty1
  • Posts: 1
  • Joined: Jan 07, 2017
|
#31929
I am completely lost as to why answer choice E is correct for this question. I went with answer B, which I didn't feel super strong about, but it still seems to make more sense than choice E. What does the area surrounding Granville have to do with the argument??
 Kristina Moen
PowerScore Staff
  • PowerScore Staff
  • Posts: 230
  • Joined: Nov 17, 2016
|
#31958
Hi mbeatty,

Welcome! This question features causal reasoning. The author is trying to make us believe that car accidents could be reduced if school began later. He offers as evidence one example - that when Granville's school day began later, car accidents were reduced.

With any causal argument, you have several ways to strengthen the causal relationship. One way you can strengthen a causal relationship is by showing that where the cause did not occur, the effect did not occur. That's answer choice (E)! In the areas surrounding Granville (where presumably the school time did not change), car accidents actually increased.

You can also think of it this way: If someone told you that accidents decreased at the same time that the school time changed, you might be thinking "Yeah, but how do we know the accidents wouldn't have decreased anyway? Maybe that's the overall trend and it had nothing to do with the school time being changed?" That would be a perfectly acceptable response! So answer choice (E) combats that complaint. And so it strengthens the argument.
 Ceci
  • Posts: 2
  • Joined: Jan 12, 2017
|
#32048
Hi!

Could you please explain why answer choice D is incorrect? It seems to demonstrate that the change in school time was effective by showing that the accidents involving teenage drivers in Granville occur primarily at night rather than in the morning.

Thank you!
User avatar
 Jonathan Evans
PowerScore Staff
  • PowerScore Staff
  • Posts: 726
  • Joined: Jun 09, 2016
|
#32051
Complete Question Explanation

Strengthen. The correct answer choice is (E)

This is an excellent example of a "defender" Strengthen question and a good illustration of the manner in which Strengthen questions can add an additional layer of complexity to what might have otherwise been easier Weaken questions. In the stimulus, the editorial concludes that postponing the beginning of high school could reduce car accidents involving teenagers. The premises offered include (1) the fact that teenagers have a biological mechanism that causes them to sleep later, (2) are sleepy if woken up too early, and (3) this sleepiness can impair driving ability. Finally, as evidence the editorial cites (4) the results of a change in Granville's high school schedule: When Granville moved the start time back to 8:30, teenage accidents declined.

Since the editorial attempts to establish a causal link between Granville's change and the decline in accidents, as a prephrase, you should anticipate an answer that likely mitigates one of the many possible causal flaw weaknesses. In other words, in this scenario, as an initial step, you should consider all the ways one might WEAKEN this causal conclusion. The likely defender credited response will mitigate one (or more) of these issues.

Answer choice (A): This choice attempts to muddle the issue by making a contrast between teenagers and young children. The reasoning behind this incorrect response might be that test-takers could conclude that an earlier school start time for younger children might be appropriate, but such a start time is inappropriate for teenagers. As it stands, this answer choice is irrelevant and does nothing to link the causal connection between later start times and fewer accidents.

Answer choice (B): This choice again tries to sidestep the issue in the scenario by providing a different kind of reason why delaying the start of high school in the morning might be a good idea. This answer choice is based on a misreading of the conclusion, i.e. it is based on the misunderstanding that the editorial is primarily concerned with advocating for a later school start time instead of attempting to show a link between the later start time and a reduction in accidents involving teenagers.

Answer choice (C): This answer choice actually weakens the conclusion by providing evidence that a delay in high school start time may not have the impact the editorial supposes since many teenagers who work would be unaffected by such a delay. Presumably, these other teenagers will continue to need to be at work at the same time regardless of what high schools do.

Answer choice (D): This answer choice is an attractive wrong answer by bringing in the issue of teenage accidents at night. It is an invitation to introduce additional assumptions, specifically that the change in school start time will be insignificant because "many" teenage accidents occur at night. There are a couple issues: (1) "many" is a nebulous quantity statement that implies "some" accidents occur at night. It does not preclude or even take away from the possibility that a significant number of accidents occur in the morning and could be lessened by a later school start time. Further, (2) this information is largely irrelevant to the matter at hand; even if "most" or "almost all" accidents involving teenagers occurred in the evening, it could still be possible that a later school start time could "reduce" teenage car accidents driving to school.

Answer choice (E): This is the correct answer choice. This answer might be clearer if you apply a quick version of the Variance Test™. Imagine that there was a steep decline in the car accidents in all the areas surrounding Granville. If this had been the case, then there would have likely been another cause for the decline in teenage car accidents in Granville (assuming that the surrounding areas did not institute a policy identical to Granville's). Such information would decidedly weaken the conclusion. The information in this answer choice is a defender against this possibility and mitigates this potential weakness.
User avatar
 Jonathan Evans
PowerScore Staff
  • PowerScore Staff
  • Posts: 726
  • Joined: Jun 09, 2016
|
#32052
Ceci, I hope the above explanation answers your question. Please let us know if you need further clarification. Thanks!
 dtodaizzle
  • Posts: 14
  • Joined: Dec 13, 2016
|
#32247
I agree with the explanations for (E). However, how do we know that the region surrounding Granville did not also push back the start time to 8:30 AM? This is a huge assumption to make. On prior LR questions that I have done, the LSAT is reasonable when asking you to make assumptions. (IE: If you commit murder, it is bad.)

Then again, this is the least bad answer.
 Kristina Moen
PowerScore Staff
  • PowerScore Staff
  • Posts: 230
  • Joined: Nov 17, 2016
|
#32268
dtodaizzle,

We don't. However, this is a Strengthen question, which means that the correct answer just has to strengthen the argument a tiny bit. For example, I could strengthen a study by saying that the data was gathered correctly. Not too much persuasion there, but it strengthens the study a tiny bit. :-D
 MikeJones
  • Posts: 31
  • Joined: Oct 02, 2017
|
#41890
Jonathan Evans wrote:Complete Question Explanation

Answer choice (E): This is the correct answer choice. This answer might be clearer if you apply a quick version of the Variance Test™. Imagine that there was a steep decline in the car accidents in all the areas surrounding Granville. If this had been the case, then there would have likely been another cause for the decline in teenage car accidents in Granville (assuming that the surrounding areas did not institute a policy identical to Granville's). Such information would decidedly weaken the conclusion. The information in this answer choice is a defender against this possibility and mitigates this potential weakness.
Fantastic explanation here. :-D
 Sophia123
  • Posts: 43
  • Joined: Mar 20, 2017
|
#47935
Hi,

This is another question where I was a little confused on where to draw the line on whether or not it was acceptable to make an assumption regarding the answer choice. For E, we do have to assume that the surrounding areas did NOT institute the same later start time policy as Granville. If they did institute the same policy, and accidents increased, that would severely weaken the argument (in that it would show cause --> no effect). However, if you assume the surrounding areas did NOT institute that policy, that does strengthen the conclusion by showing no cause --> no effect.

So my question is - when it is it appropriate to help the answer choice with additional assumptions?

Thanks in advance!

Best,
Sophia
 Adam Tyson
PowerScore Staff
  • PowerScore Staff
  • Posts: 5153
  • Joined: Apr 14, 2011
|
#48936
actually, Sophia, we don't have to make that assumption! Instead, we can look at answer choice E as Kristina and Jonathan did in their explanations above as defending against the possibility that the improvement in Granville was due to a larger trend. Don't many make any additional assumptions! None are needed here.

Get the most out of your LSAT Prep Plus subscription.

Analyze and track your performance with our Testing and Analytics Package.